LSAT and Law School Admissions Forum

Get expert LSAT preparation and law school admissions advice from PowerScore Test Preparation.

 Administrator
PowerScore Staff
  • PowerScore Staff
  • Posts: 8917
  • Joined: Feb 02, 2011
|
#23761
Complete Question Explanation

Method of Reasoning-SN. The correct answer choice is (B)

The stimulus attacks the belief that the Premier will reshuffle the cabinet, concluding that the belief is probably false, because in the past every reshuffling was preceded by certain meetings, but no such meetings have recently occurred or been planned.

The stimulus is flawed because it involves assuming that two events were associated even though they could have been coincidental, and presumes that meetings associated with the shufflings would probably be a necessary condition for shuffling, even though many other means of preparation could exist.

You are asked to identify a principle of reasoning (similar to a method of reasoning), so you should focus on the flaws that the stimulus commits.

Answer choice (A): Since the argument attacks the rumors rather than supports them, it is difficult to interpret this incorrect choice as consistent with the stimulus. Furthermore, the stimulus references no premises that support the rumors. If you assumed that this principle governs its own conclusion, and that the conclusion is true, you should still eliminate this choice, because the conclusion is probabilistic even though the premises are certain, but this choice claims that certain premises should be followed by a certain conclusion.

Answer choice (B): This is the correct answer choice, and is worded correctly though archaically. The principle is that since the author has come to expect that a shuffling should be preceded by certain meetings, when the meetings do not exist, that undermines the hypothesis that a shuffling will occur. The author does not bother with the possibility that unexpected situations could also involve shuffling the cabinet.

Answer choice (C): The stimulus argues that the hypothesis that the Premier will reshuffle the cabinet is unsupported by the current situation, not that the hypothesis could be false even though it is supported. This choice is wrong.

Answer choice (D): This response cannot be correct for two reasons, because it describes a correct principle similar, but not identical, to the one that the stimulus ignores. The stimulus ignored that the possibility that meetings are only one method from a few, any of which could satisfy the necessary condition for shuffling the cabinet. This response is about causal reasoning, and correctly states that a usual cause does not have to be the necessary cause.

Answer choice (E): Since the stimulus is based on showing that the rumors are probably false, it does not make sense to conclude that it is based on a principle that is geared toward uncertainty (“it cannot be deduced…”), so you can easily eliminate this response if you avoid attempting to interpret it in entirety. Alternatively, the argument is that since meetings are probably necessary for shuffling the cabinet, a lack of meetings is inconsistent with the rumors that the Premier will reshuffle the cabinet, so those rumors are probably false. Concluding that something cannot be determined to be true is not the same as concluding that something is probably false, even if you do believe that the bulk of this choice describes the reasoning in the stimulus fairly well.
 lathlee
  • Posts: 652
  • Joined: Apr 01, 2016
|
#42697
I got this question correct, but just making it sure, isn't this Question type Method - Principle with SN?
 Adam Tyson
PowerScore Staff
  • PowerScore Staff
  • Posts: 5153
  • Joined: Apr 14, 2011
|
#42727
I think you could view this as a Principle question as much as a Method of Reasoning question. We are tasked with figuring out which rule or guideline (Principle) the author used to get to his conclusion (Method). The label shouldn't do anything to change your approach in this case, but I think you are correct to consider that this question is as much about the principle as it is about the method.

Good work!
User avatar
 sjlsat
  • Posts: 8
  • Joined: Jan 24, 2023
|
#100033
Administrator wrote: Mon May 02, 2016 1:38 pm Complete Question Explanation

Method of Reasoning-SN. The correct answer choice is (B)

Answer choice (A): Since the argument attacks the rumors rather than supports them, it is difficult to interpret this incorrect choice as consistent with the stimulus. Furthermore, the stimulus references no premises that support the rumors. If you assumed that this principle governs its own conclusion, and that the conclusion is true, you should still eliminate this choice, because the conclusion is probabilistic even though the premises are certain, but this choice claims that certain premises should be followed by a certain conclusion.

Hello, I picked answer choice A for this one but I’m just having a little bit of trouble understanding the above - three questions.

1. If the conclusion was in favor of the rumors, for example, that the rumors were most likely true, would this conclusion follow “logically from a set of premises?” Or would you need a premise that specifically indicates either position towards the conclusion (a premise that is either in favor or against)?

2. Do “follow logically from the set of premises” answer choices usually only refer to stimuli where there is no conclusion that is a subjective opinion? As in mostly true or mostly false vs smt that is more concrete?

3. If the premise in here about every reshuffle in the past having a meeting before it was changed to something more definite, like “The Premier always has a planned meeting before a reschedule per X rule,” then would answer choice A make more sense? Is it just the fact that we can’t make the assumption that even if two things that were always associated with each other happened in the past, they can’t happen in the present unless we’re told so?

Thank you!!!!!
 Robert Carroll
PowerScore Staff
  • PowerScore Staff
  • Posts: 1787
  • Joined: Dec 06, 2013
|
#100473
sjlsat,

You should seek the referent of "a conclusion". What conclusion follows logically from a set of premises? The only conclusion at all in the stimulus is the conclusion of the author himself. The "rumors" aren't presented as a conclusion by anyone - certainly the author thinks the rumors are most likely false, but even those believing in the rumors, whoever they are, don't have any evidence for them of which we are aware in the stimulus. So the only conclusion whatsoever is the last sentence. Answer choice (A) is then saying something like "If I have already shown that my conclusion follows logically from my premises, then that proves that my conclusion cannot be less likely than my premises." This doesn't seem like something the author cares about. They care about proving their conclusion. If they prove that, they don't really have to go on to say "and therefore, my conclusion is at least as likely as my premises." Once the conclusion is proven (in the language of answer choice (A), "a conclusion follows logically from a set of premises"), there doesn't seem to be anything more the author wants to prove. Instead their argument is relying on an expectation of a certain event's occurrence in certain circumstances. Because the event did not occur, the circumstances are probably not present.

To your second point, a conclusion can logically follow from premises even if the conclusion includes a word like "likely". That lesser degree of certainty does not present the conclusion's logically following. In fact, it really can't - if a more certain conclusion would have followed, then certainly a lesser one would even more strongly. Consider the following argument:

In order to receive the scholarship, you need a GPA above 3.5.

Your GPA is 3.4.

Therefore, you cannot receive the scholarship.

The conclusion follows logically from the premises. Now change the conclusion to the following:

Therefore, you likely will not receive the scholarship.

That conclusion still follows logically. Thus, following logically is something that can happen even with less-than-certain conclusion.

Further, "most likely false" is not a subjective opinion. It's a fact-type statement.

To your third point, that change would not change anything about the incorrectness of answer choice (A).

Robert Carroll
User avatar
 jailenea
  • Posts: 25
  • Joined: Aug 30, 2021
|
#101773
I can see why B is correct, but I fell for answer A and am having trouble breaking part with it.

The way I understood answer A, as applied to the question, is as follows:

When a conclusion (the rumors are likely false) follows logically from a set premises (no meetings between Premier and sr cabinet members), the probability that the conclusion is true (meaning it's true the rumors are likely false) cannot be any less true than the probability that the premises (no such meetings before the reshuffle) are all all true.

Although, in writing that, I question my own thought process because of the latter half. Specifically, "...the probability that the conclusion is true (meaning it's true the rumors are likely false) cannot be any less true than the probability that the premises (no such meetings before the reshuffle) are all all true." Since the conclusion is given in terms of likelihood (but the premises are not), then the probability the conclusion is true is in fact less than the premises. That makes A incorrect.

Please let me know if this is a good way to go about it or if I am missing something.
 Jeremy Press
PowerScore Staff
  • PowerScore Staff
  • Posts: 1000
  • Joined: Jun 12, 2017
|
#101776
Hi jailenea,

You're correct that the second half of answer choice A would be problematic for the reason you've identified. But I'd also question the first half of answer choice A. For a conclusion to "follow logically" from certain premises, it has to be irresistible based on those premises. This conclusion doesn't meet that standard. It's not irresistibly true that the rumors are probably false purely based on the lack of occurrence of meetings. That's because the present/future doesn't necessarily have to match the past, and we don't have enough other information to predict the precise level of likelihood that this rumor is false.

In short, be careful about language that says a conclusion "follows logically" from its premises. That's a very strong statement and is very rarely satisfied by a stimulus argument on this test!

Get the most out of your LSAT Prep Plus subscription.

Analyze and track your performance with our Testing and Analytics Package.